Answer:
Step-by-step explanation:
Which statements about the design are true? select three options. each sprinkler head is placed more than 3 feet from the walkway. if two additional sprinkler heads are placed on the lawn, every square foot of the yard would be watered. the total area watered by each sprinkler is approximately 63.6 square feet. the total area watered by all the sprinklers is approximately 254 square feet. if the sprinklers were placed in two straight lines on the yard, they would water a greater amount of the lawn.
The true statements are:
A. Each sprinkler head is placed more than 3 feet from the walkway.C. The total area watered by each sprinkler is approximately 63.6 square feet.D. The total area watered by all the sprinklers is approximately 254 square feet.Calculations and Parameters:Given that:
Sprinkler head = More than one grid box awayEach grid= 3 feetSprinkler distance= 1.5 boxesRadius=?Area=?To find the radius, 3 * 1.5= 4.5 feet.
The area of each circle=
[tex]3.15(4.5)^2 \\= 63.585 \\ 63.6 ft^2.[/tex]
Hence, the statements that are true are options A, C, and D.
Read more about area of a circle here:
https://brainly.com/question/6943527
#SPJ4
Answer:
It is A C D
Step-by-step explanation:
:) good luck
− 96 , 48 , − 24 , 12 , − 6 . what is the ratio of the geometric sequece
Answer:
-1/2
Step-by-step explanation:
it has to be a negative number since
every other number is negative
its a fraction since the numbers are decreasing
96 divided by 48 is 2
so the number being multiplied or ration must be -1/2
Find the missing term
(m-n)^2=m^2-_____+n^2
Answer:
2mn
Step-by-step explanation:
The expanded form of the square can be found using the distributive property.
__
(m -n)² = (m -n)(m -n) = m(m -n) -n(m -n) . . . use the distributive property
= m² -mn -nm +n² . . . . use it again
= m² -2mn +n² . . . . . . combine like terms
The missing term is 2mn.
A straight angle equals 180 degrees is true or false?
Answer:
True
Step-by-step explanation:
Straight angles are straight lines which equal 180 degrees
Answer:
true it is equals 180 for that angle
If 25% of the people in a small town are voters and there are 2360 voters, what is the population of the town?
Answer:
x=11800
Step-by-step explanation:
make a proportion:
2360/.2=X/1
2360(1)=.2x
x=11800
math hw 2
The Binomial Distribution
pls help me with my homework!!
thanks,
Using the binomial distribution, we have that:
a) The distribution is given by:
P(X = 0) = 0.0625.P(X = 1) = 0.25.P(X = 2) = 0.375.P(X = 3) = 0.25.P(X = 4) = 0.0625.b) 0.3125 = 31.25%.
What is the binomial distribution formula?The formula is:
[tex]P(X = x) = C_{n,x}.p^{x}.(1-p)^{n-x}[/tex]
[tex]C_{n,x} = \frac{n!}{x!(n-x)!}[/tex]
The parameters are:
x is the number of successes.n is the number of trials.p is the probability of a success on a single trial.In this problem, we have that:
There are 4 children, hence n = 4.Each children has a 50% chance of being female, hence p = 0.5.The distribution is given by the probability of each outcome, hence:
[tex]P(X = x) = C_{n,x}.p^{x}.(1-p)^{n-x}[/tex]
[tex]P(X = 0) = C_{4,0}.(0.5)^{0}.(0.5)^{4} = 0.0625[/tex]
[tex]P(X = 1) = C_{4,1}.(0.5)^{1}.(0.5)^{3} = 0.25[/tex]
[tex]P(X = 2) = C_{4,2}.(0.5)^{2}.(0.5)^{2} = 0.375[/tex]
[tex]P(X = 3) = C_{4,3}.(0.5)^{3}.(0.5)^{1} = 0.25[/tex]
[tex]P(X = 4) = C_{4,4}.(0.5)^{4}.(0.5)^{0} = 0.0625[/tex]
The probability of at least 3 daughters is given by:
[tex]P(X \geq 3) = P(X = 3) + P(X = 4) = 0.25 + 0.0625 = 0.3125[/tex].
More can be learned about the binomial distribution at https://brainly.com/question/24863377
#SPJ1
three times a number increased by 2 is greater than zero or less than or equal to negative three
Step-by-step explanation:
Let the number be X
3×X=3X
Increased by 2
3X+2>0≤-3
A pattern has 4 blue triangles to every 310 yellow triangles. What is the ratio of yellow triangles to all triangles?
Answer:
blue to all triangles ratio: 1 : 78.5
Step-by-step
so first the total amount of triangles is 4 blue triangles to 314 total triangle or 4/314
you could just leave it like that or simplify it by dividing both sides by four getting 1 : 78.5
a book store was selling 7 books for $39.55. online you could buy 3 books for $24.96. which place has a lower unit price
Answer:
the book store selling 7 books for $39.55 (the book store)
Step-by-step explanation:
we have to calculate what each book costs. to find out what each book costs, we have to use division...
so for the book store:
39.55 / 7 = 5.65 - this means that each book costs $5.65 by itself
for the online store:
24.96 / 3 = 8.32 - this means that each book costs $8.32 by itself
now we have to compare the prices... $5.65 costs less than $8.32, so the book store would have a lower unit price.
what's the answer to 4x-6=3x
Complete this statement:
10x^3a+6xa^2=2xa( )
Answer:
2xa(5x^2 + 3a).
Step-by-step explanation:
We are factorising:-
Divide the left side by 2xa and place the result in the parentheses:
10x^3a+6xa^2 = 2xa(5x^2 + 3a )
Answer:
2xa(5x² +3a-1)
Step-by-step explanation:
10x³a+ 6xa²-2xa=2xa-2xa
10x³a+ 6xa²-2xa=0
Factor 10x³a+ 6xa²-2xa:
2ax(5x² + 3a -1)
Actually 100 points check for yourself (Easiest 100 points) Match each ordered pair on the left with the functions on the right.
Answer:
(-4, 40) y = -9x + 4
(8, 50) y = 6x + 2
(9, -36) y = -3x - 9
(5, 38) y = 8x - 2
Do you want an explanation?
#1
y=-9x+4Put (-4,40)
40=-9(-4)+440=40Verified
#2
y=-3x-9put (9,-36)
-36=-3(9)-9-36=-36Verified
#3
y=8x-2Put (5,38)
38=8(5)-238=38Verified
#4
y=6x+2Put (8,50)
50=6(8)+250=50Hence verifiedl
What type of sampling is described in this study?
A company that sells hair-care products wants to
determine if a product that combines shampoo and
conditioner works better than a shampoo and
conditioner used separately. A researcher recruits 60
volunteers and pairs them according to age, hair color,
and hair type. For each pair, the researcher flips a coin
to determine which volunteers will use the
shampoo/conditioner combination and which ones will
use the separate shampoo and conditioner. After using
the products for one month, the subjects are asked to
rate their satisfaction with the hair products on a scale
of 1-10 (1 = highly dissatisfied and 10 = highly
satisfied). What type of sampling is described in this study?
one sample
paired data
two samples
more than two samples
The sampling type in this study, where a researcher recruits 60 paired volunteers and asks them to rate their satisfaction after product use, is B. paired data.
What is a paired data test?A paired data test or sampling is used when the researcher is interested in the difference between two variables for the same subject.
For instance, in this experiment, the researcher wants to find the satisfaction level of the volunteers who used the shampoo and conditioner separately or combined.
Thus, the sampling type in this study, where a researcher recruits 60 paired volunteers and asks them to rate their satisfaction after product use, is B. paired data.
Learn more about paired data at https://brainly.com/question/17438514
#SPJ1
1
math
math geniuses pls help!!!
pls check other questions as well thanks,!
Using the given box and whisker plot, it is found that the mean most likely exceeds the median for Elaine, hence option B is correct.
What does a box and whisker plot shows?A box and whisker plot shows three things:
The 25th percentile, which is the median of the bottom 50%.The median.The 75th percentile, which is the median of the upper 50%.In this problem:
The plot for Sarah is right-skewed, due to the proximity of the 25th percentile and the median. In a right skewed plot, the mean is less than the median.For Elaine, it is left-skewed, due to the proximity of the median to the 75th percentile. In a left skewed plot, the mean exceeds the median.The mean most likely exceeds the median for Elaine, hence option B is correct.
More can be learned about box and whisker plots at https://brainly.com/question/27743205
#SPJ1
The mean number wins of a team is 37 wins that is normally distributed. The standard deviation is 3 wins. What would be the sections that are one standard deviation from the mean? Select all that apply.
31
34
37
40
43
Using the normal distribution, it is found that the sections that are one standard deviation from the mean are 34 and 40.
Normal Probability DistributionThe z-score of a measure X of a normally distributed variable with mean [tex]\mu[/tex] and standard deviation [tex]\sigma[/tex] is given by:
[tex]Z = \frac{X - \mu}{\sigma}[/tex]
The z-score measures how many standard deviations the measure is above or below the mean. Looking at the z-score table, the p-value associated with this z-score is found, which is the percentile of X.The mean and the standard deviation are given, respectively, by:
[tex]\mu = 37, \sigma = 3[/tex]
The measures that are one standard deviation from the mean are given by X when Z = -1 and Z = 1, hence:
[tex]Z = \frac{X - \mu}{\sigma}[/tex]
[tex]-1 = \frac{X - 37}{3}[/tex]
X - 37 = -3
X = 34
[tex]Z = \frac{X - \mu}{\sigma}[/tex]
[tex]1 = \frac{X - 37}{3}[/tex]
X - 37 = 3
X = 40
More can be learned about the normal distribution at https://brainly.com/question/24663213
#SPJ1
The perimeter of a rectangle is 90 feet. The width is 15 feet. What is the perimeter?
A ball is dropped from a height of 600 feet. The function describing the height of the ball at t seconds after it dropped is [tex]f(t)=-16t^2+600[/tex].
a) Find the average velocity of the object during the first 3 seconds.
b) Verify that at some time during the first 3 seconds the instantaneous velocity equals the average velocity. Find that time.
The average velocity: _ ft/sec
The instantaneous velocity equals to the average velocity at t = _ sec
The average velocity of the object after during the first three seconds is: 48m/s
The time at which the instantaneous velocity equals the average velocity within the first three seconds is 1.5 seconds.
What is instantaneous and average velocities?Instantaneous velocity is the speed of an object at a particular point in time.
Average velocity is the velocity of an object after covering a certain distance for a period of time
Analysis:
Given
initial height = 600 feet
Height with respect to time = f(t) = -16[tex]t^{2}[/tex] + 600
a) Height at t = 0 = 600 feet
Height at t = 3 seconds = f(3) = -16[tex](3)^{2}[/tex] + 600 = 456 feet
Distance travelled = 600 - 456 = 144 feet
Average velocity = distance travelled/time taken = 144/3 = 48 feet/seconds
b) instantaneous velocity at time t = [tex]\frac{df(t)}{dt}[/tex] = [tex]\frac{d(-16t^{2} + 600) }{dt}[/tex] = -32t
when instantaneous velocity equal average velocity
-32t = -48
t = 1.5 seconds
In conclusion, the Average velocity after 3 seconds is 48 feet per seconds and the time taken for the average velocity to equal the instantaneous velocity is 1.5 seconds.
Learn more about instantaneous and Average velocity: brainly.com/question/13372043
#SPJ1
There is a pair of parallel sides in the following shape units^2
Parallel lines are the top & bottom lines - labelled 10 & 2
A = 0.5(2+10) x 2
=12 units^2
Hope this helps!
Ali bought 212 pounds of ground turkey at $2.50 per pound and 314 pounds of ground beef at $3.20 per pound.
What was her total cost?
Enter your answer in the box.
Answer:
Step-by-step explanation:
the total cost for this question is $12.10.
let me know if this helped.
please solve this problem using the pythagorean theorem, and give me the right answer and I will give you brainliest. :)
======================================================
Explanation:
See the diagram below.
The diagram shows we have a right triangle with these side lengths:
a = 8 metersb = 5.5 metersc = unknown hypotenuseWe'll use the pythagorean theorem to find c
[tex]a^2 + b^2 = c^2\\\\c = \sqrt{a^2 + b^2}\\\\c = \sqrt{(8)^2 + (5.5)^2}\\\\c = \sqrt{94.25}\\\\c \approx 9.7082\\\\c \approx 9.7\\\\[/tex]
Mrs. Durham and Brett are about 9.7 meters apart.
Help on this math a s a p if possible
Answer:
Mean - 17.222222222222
Median - 17
Mode - 12
Step-by-step explanation:
lets arrange this is least to greatest.
2, 4, 12, 12, 17, 18, 28, 29, 33
to find the mean(even though you dont need it), you add all the numbers up and devide by the amount of numbers.
so -> 2 + 4 + 12 + 12 + 17 + 18 + 28 + 29 + 33 = 155
and there are 9 numbers, so 155 / 9 = 17.2 and a lot more 2s
to find the median, you arrange from least to greatest vice versa, then find the middle number.
2, 4, 12, 12, 17, 18, 28, 29, 33
the middle number is 17, so the mean is 17
to find the mode, you have to find the number that is most repeated.
2, 4, 12, 12, 17, 18, 28, 29, 33
the number that is repeated the most is 12, so your mode is 12.
Hope this helped!!!
Can someone help me I dont know what to do
Step-by-step explanation:
1. find P of semicircle
perimeter of a semicircle = pi×r+d
=3.14×5+10
=25.7
2.find P of rectangle
P=10×4=40cm
3. add P of semicircle and P of rectangle
25.7+40=65.7cm
A 6-ounce jar of modeling clay costs $4.20. What is the unit price?
Answer:
$0.70 per ounce
Step-by-step explanation:
Concepts + Application
The unit price of an object, in this case, is the total amount of money spent divided by how many ounces there are. We're given the total amount of money as $4.20 and the number of ounces as 6. Thus, we just divide 4.20 by 6.
Solution
4.20 ÷ 60.70$0.70 per ouncewhat would be 3/4 of 960.
Answer:
720
Step-by-step explanation:
Three-fourths of 960 are simply three fourths times 960, which can be written as follows:
Three/fourths x 960
Furthermore, you can convert "three" to "3" and "fourths" to "4" and then the equation and answer are:
3/4 x 960 = 720
A grandson's age in months is equal to his grandfather's age in years. The difference
of the grandson's age in years and the grandfather's age in years is 77. Find the
grandson's age in years.
The difference of the grandson's age in years and the grandfather's age in years is 77. Therefore, the age of the grandson is 7 years.
How to use equations to find the age?
A grandson's age in months is equal to his grandfather's age in years.
Recall,
12 months = 1 year
Hence,
let
x = age of grandson in month
It is the same age in years for the grandfather.
Therefore,
x = age of grand father in years
The difference of the grandson's age in years and the grandfather's age in years is 77. Therefore,
x - x / 12 = 77
12x - x/12 = 77
11x / 12 = 77
cross multiply
11x = 924
x = 924 / 11
x = 84
Therefore, the grandson age in in years is 84 / 12 = 7 years.
learn more on equation here: https://brainly.com/question/11136531
#SPJ1
please help show work
Answer:
236854Step-by-step explanation:
Relation between sugar and flour
18 teaspoons of sugar per 3 cups of flourRate : 18/3 = 6 teaspoons of sugar per cup of flourTable
[tex]\left[\begin{array}{ccc}sugar(tsp)&flour(cups)\\12&2\\18&3\\36&6\\48&8\\54&9\end{array}\right][/tex]
Answer:
Step-by-step explanation:
Remark
The key to this question is the second entry. Numerically the sugar is divided by 6 to get the flour. Never mind the units. Just deal with the ratio itself.
Formula
Flour = Sugar / 6
Solution
Sugar = 12
Flour = 12/6 = 2
Flour = 18/6
Flour = 3
Flour = 6
6 = sugar/6 multiply by 6
6*6 = 6* sugar/6
36 = sugar
Sugar = 48
Flour = 48/6 = 8 cups of flower
Flour = 9
9 = sugar / 6
9*6 = sugar
Sugar = 54.
I’ll give brainliest please help! Explain the Polynomial equation using Long division
Answer:
The solution is in the image
Mrs. Purdue needs to pick three random students in her class to be representatives on the student council. Explain how Mrs. Purdue could use slips of paper to generate a random sample.
Answer:
See below
Step-by-step explanation:
She could have each student in the class write his or her name on a slip of paper and put it in a bowl. Then, she could shake the bowl and pull out any slip of paper with a student's name on it. After 3 times, the random sample would be over, and Mrs. Purdue would have her 3 representatives.
i need help with this question
An airplane is flying at an altitude of 2.7 miles and is 8.3 miles from the runway. Find the angle of depression from the airplane to the runway
Answer:
Step-by-step explanation:
AB = altitude = 2.7 miles
BC = 8.3 miles
[tex]\sf tan \ \theta = \dfrac{opposite \ side \ of \ \theta}{adjacent \ side \ of \ \theta}\\\\Tan \ \theta = \dfrac{AB}{BC}=\dfrac{2.7}{8.3}\\\\Tan \ \theta = 0.33\\\\\theta = tan^{-1} \ 0.33[/tex]
[tex]\boxed{\theta = 18.3}[/tex]